MEDSURG II: Saunders Emergency

Pataasin ang iyong marka sa homework at exams ngayon gamit ang Quizwiz!

A mother brings her child to the emergency department. Based on the child's sitting position, drooling, and apparent respiratory distress, a diagnosis of epiglottitis is suspected. In anticipation of the health care provider's prescriptions, in which order of priority would the nurse implement the actions? Arrange the actions in the order that they should be performed. All options must be used. 1.Maintain a patent airway. 2.Assess breath sounds by auscultation. 3.Obtain an oxygen saturation level using pulse oximetry. 4.Insert an intravenous line for fluid administration. 5.Obtain an axillary temperature. 6.Administer an antipyretic.

1.Maintain a patent airway. 2.Assess breath sounds by auscultation. 3.Obtain an oxygen saturation level using pulse oximetry. 4.Insert an intravenous line for fluid administration. 5.Obtain an axillary temperature. 6.Administer an antipyretic. Rationale: Maintenance of a patent airway is essential. Emergency intubation equipment should be readily available. Once a patent airway is determined or established and maintained, the breath sounds should be auscultated. This action is followed by checking the pulse oximetry. All of these interventions relate to the respiratory status and are the priority. Following this, fluid needs should be considered because their administration relates to maintaining circulatory status. Then the temperature is taken and an antipyretic is administered (usually by the rectal route) if needed.

The nurse is caring for a client diagnosed with cirrhosis of the liver with portal hypertension. The client vomited 500 mL bright red emesis and states that he is feeling lightheaded. In which priority order should the nurse perform these interventions? Arrange the actions in the order they should be performed. All options must be used. 1. Apply oxygen. 2. Check the client's blood pressure. 3. Ensure that 2 large-bore intravenous lines are present with an isotonic solution infusing. 4. Ask the client if he is taking any nonsteroidal antiinflammatory medications.

Correct Answers: 1.Apply oxygen. 1 2.Ensure that 2 large-bore intravenous lines are present with an isotonic solution infusing. 2 3.Check the client's blood pressure. 3 4.Ask the client if he is taking any nonsteroidal antiinflammatory medications. 4 Rationale: The client has an upper gastrointestinal (GI) bleed. Upper GI bleeding is an emergency because it can lead to hypovolemic shock. The first intervention of those listed should be to apply oxygen in an attempt to maximize the amount of oxygen being delivered by the decreased number of red blood cells due to the bleeding. The next action should be to ensure that 2 large-bore intravenous (IV) lines are present, and begin replacement of the intravascular fluid volume with an isotonic IV fluid. The nurse should then check the blood pressure. These are all actions to stabilize and assess the client's current condition. The last intervention is to ask the client about nonsteroidal antiinflammatory medications. Although it is important to identify the cause of the bleeding and obtain a complete history of events leading up to the bleeding episode, this needs to be deferred until emergency care is initiated.

The nurse is monitoring a client receiving total parenteral nutrition (TPN). The client suddenly develops respiratory distress, dyspnea, and chest pain, and the nurse suspects air embolism. In order of priority, how should the nurse plan the actions to take? Arrange the actions in the order that they should be performed. All options must be used. 1. Administer oxygen. 2. Document the occurrence. 3. Take the client's vital signs. 4. Clamp the intravenous (IV) catheter. 5. Contact the health care provider (HCP). 6. Position the client in a left Trendelenburg's position.

Correct Answers: 1.Clamp the intravenous (IV) catheter. 2.Position the client in a left Trendelenburg's position. 3.Contact the health care provider (HCP). 4.Administer oxygen. 5.Take the client's vital signs. 6.Document the occurrence. Rationale: Air embolism occurs when air enters the catheter system during IV tubing changes or when the IV tubing disconnects. Air embolism is a critical situation. If air embolism is suspected, the nurse would first clamp the IV catheter to prevent further introduction of air and the air embolism from traveling through the heart to the pulmonary system. The nurse would next place the client in a left side-lying position with the head lower than the feet (to trap air in the right side of the heart). The nurse would notify the HCP and administer oxygen as prescribed. The nurse would monitor the client closely and take the client's vital signs. Finally, the nurse documents the occurrence.

A client with a spinal cord injury suddenly experiences an episode of autonomic dysreflexia. After checking the client's vital signs, the nurse takes the following actions. Arrange the actions in the order they should be performed. All options must be used. 1. Raise the head of the bed. 2. Loosen tight clothing on the client. 3. Check for bladder distention. 4. Contact the health care provider (HCP). 5. Administer an antihypertensive medication. 6. Document the occurrence, treatment, and response.

Correct Answers: 1.Raise the head of the bed. 2.Loosen tight clothing on the client. 3.Check for bladder distention. 4.Contact the health care provider (HCP). 5.Administer an antihypertensive medication. 6.Document the occurrence, treatment, and response. Rationale: Autonomic dysreflexia is characterized by severe hypertension, bradycardia, severe headache, nasal stuffiness, and flushing. The cause is a noxious stimulus, most often a distended bladder or constipation. Autonomic dysreflexia is a neurological emergency and must be treated promptly to prevent a hypertensive stroke. Immediate nursing actions are to sit the client up in bed in a high Fowler's position and to remove the noxious stimulus. The nurse would loosen any tight clothing and then check for bladder distention. If the client has a Foley catheter, the nurse would check for kinks in the tubing. The nurse also would check for a fecal impaction and would disimpact the client, if necessary. The HCP is then contacted, especially if these actions do not relieve the signs and symptoms. Antihypertensive medication may be prescribed by the HCP to minimize cerebral hypertension. Finally, the nurse documents the occurrence, treatment, and client response.

A client is diagnosed with atrial fibrillation, and the health care provider prescribes medication therapy. Prior to initiating medication therapy, which cardiac rhythm would the nurse expect to note on the cardiac monitor? Click on the image to indicate your answer.

Rationale: In atrial fibrillation, multiple rapid impulses from many foci depolarize in the atria in a totally disorganized manner. The atria quiver, which can lead to thrombus formation. Usually no definitive P waves can be observed, only fibrillatory waves before each QRS. Medication therapy is often effective for treating atrial fibrillation. In normal sinus rhythm (option 1), atrial and ventricular rhythms are regular and the rates are 60 to 100 beats per minute. Ventricular tachycardia (option 3) occurs because of a repetitive firing of an irritable ventricular ectopic focus at a rate of 140 to 250 beats/minute or more; it can lead to cardiac arrest. In ventricular fibrillation (option 4), impulses from many irritable foci in the ventricles fire in a totally disorganized manner. It is characterized by a chaotic rapid rhythm in which the ventricles quiver and there is no cardiac output. The client lacks a pulse, blood pressure, respirations, and heart sounds. Ventricular fibrillation is fatal if not successfully terminated within 3 to 5 minutes.

A 2-year-old child is being transported to the trauma center from a local community hospital for treatment of a burn injury that is estimated as covering more than 40% of the body. The burns are both partial- and full-thickness burns. The nurse is asked to prepare for the arrival of the child and gathers supplies, anticipating that which treatment will be prescribed initially? 1. Insertion of a Foley catheter 2. Insertion of a nasogastric tube 3. Administration of an anesthetic agent for sedation 4. Application of an antimicrobial agent to the burns

Rationale: 1 A Foley catheter is inserted into the child's bladder so that urine output can be accurately measured on an hourly basis. Although pain medication may be required, the child would not receive an anesthetic agent and should not be sedated. The burn wounds would be cleansed after assessment, but this would not be the initial action. Intravenous fluids are administered at a rate sufficient to keep the child's urine output at 1 to 2 mL/kg of body weight per hour for children weighing less than 30 kg, thus reflecting adequate tissue perfusion. A nasogastric tube may or may not be required but would not be the priority intervention.

Acetylcysteine is prescribed for a client in the hospital emergency department after diagnosis of acetaminophen overdose. The nurse prepares to administer the medication using which procedure? 1. Diluting the medication in cola and administering it to the client orally 2. Calling the respiratory department to administer the medication via inhaler 3. Obtaining a 1-mL syringe to administer the small dose via the subcutaneous route 4. Obtaining an appropriate-size syringe and needle for intramuscular injection in the ventrogluteal muscle

Rationale: 1 Acetylcysteine can be given orally or by nasogastric tube to treat acetaminophen overdose. It is administered by inhalation for use as a mucolytic. Before administration of the medication as an antidote, the nurse would ensure that the client's stomach is empty through emesis or gastric lavage. The solution is diluted in cola, water, or juice to make the solution more palatable. It is not administered via nebulizer, subcutaneously, or intramuscularly for the client experiencing acetaminophen overdose.

The mother of a 3-year-old boy calls the emergency department and states that she found an empty bottle of acetaminophen on the floor. She states that she thinks her child ingested all of the medication. What is the priority question for the nurse to ask the mother? 1. "Is your child breathing okay?" 2. "Is your child alert and oriented?" 3. "Where is your child at this moment?" 4. "Do you know how many tablets were in the bottle?"

Rationale: 1 Airway is always the highest assessment data to obtain during a poison control call. Once this information is obtained, the child's neurological status can be determined in terms of his orientation and other information, such as that referred to in the remaining options.

A client is admitted to the hospital for an acute episode of angina pectoris. Which parameter is the priority for the nurse to monitor? 1. Pulse and blood pressure 2. Temperature and respirations 3. Food tolerance and urinary output 4. Right upper quadrant pain and fatigue

Rationale: 1 Angina pectoris is transient chest pain or discomfort that is caused by an imbalance between myocardial oxygen supply and demand. The discomfort typically occurs in the retrosternal area; may or may not radiate; and is described as a tight, heavy, squeezing, burning, or choking sensation. The two major types of angina pectoris are stable (classic exertional) angina and unstable angina. Stable angina, the most common type, is usually precipitated by physical exertion or emotional stress, lasts 3 to 5 minutes, and is relieved by rest and nitroglycerin. Acute intervention for the client who has an anginal attack includes vital signs, oxygen, pain relief, and continuous electrocardiographic monitoring.

The nurse is watching the cardiac monitor, and a client's rhythm suddenly changes. There are no P waves; instead, there are fibrillatory waves before each QRS complex. How should the nurse correctly interpret the client's heart rhythm? 1. Atrial fibrillation 2. Sinus tachycardia 3. Ventricular fibrillation 4. Ventricular tachycardia

Rationale: 1 Atrial fibrillation is characterized by a loss of P waves and fibrillatory waves before each QRS complex. The atria quiver, which can lead to thrombus formation.

Which should the nurse do when setting up an arterial line? 1. Tighten all tubing connections. 2. Use macrodrop intravenous tubing. 3. Level the transducer to the ventricle. 4. Raise the height of the normal saline infusion to prevent backup.

Rationale: 1 Because the arterial vasculature is a high-pressure system, all tubing connections must be tight to avoid blood loss from loose connections. High-pressure tubing with a transducer is used (not macrodrip tubing). The transducer should be level to the atrium, not the ventricle. Raising the height of the infusion is not sufficient to prevent backflow.

A client is admitted to the emergency department with chest pain that is consistent with myocardial infarction based on elevated troponin levels. Heart sounds are normal and vital signs are noted on the client's chart. The nurse should alert the health care provider because these changes are most consistent with which complication? Refer to chart below. 1. Cardiogenic shock 2. Cardiac tamponade 3. Pulmonary embolism 4.Dissecting thoracic aortic aneurysm

Rationale: 1 Cardiogenic shock occurs with severe damage (more than 40%) to the left ventricle. Classic signs include hypotension; a rapid pulse that becomes weaker; decreased urine output; and cool, clammy skin. Respiratory rate increases as the body develops metabolic acidosis from shock. Cardiac tamponade is accompanied by distant, muffled heart sounds and prominent neck vessels. Pulmonary embolism presents suddenly with severe dyspnea accompanying the chest pain. Dissecting aortic aneurysms usually are accompanied by back pain.

The emergency department nurse is monitoring a client who received treatment for a severe asthma attack. The nurse determines that the client's respiratory status has worsened if which is noted on assessment? 1. Diminished breath sounds 2. Wheezing during inhalation 3. Wheezing during exhalation 4. Wheezing throughout the lung fields

Rationale: 1 Diminished breath sounds may be an indication of severe obstruction and possibly respiratory failure. Wheezing is not a reliable manifestation to determine the severity of an asthma attack. For wheezing to occur, the client must be able to move sufficient air to produce breath sounds. Wheezing usually occurs first on exhalation. As the asthma attack progresses, the client may wheeze during both inspiration and expiration.

The nurse is reviewing the laboratory test results for a client admitted to the burn unit 3 hours after an explosion that occurred at a worksite. The client has a severe burn injury that covers 35% of the total body surface area (TBSA). The nurse is most likely to note which finding on the laboratory report? 1. Hematocrit 60% (0.60) 2. Serum albumin 4.8 g/dL (48 g/L) 3. Serum sodium 144 mEq/L (144 mmol/L) 4. White blood cell (WBC) count 9000 mm3 (9 × 109/L)

Rationale: 1 Extensive burns greater than 25% of the TBSA result in generalized body edema in both burned and unburned tissues and a decrease in circulating intravascular blood volume. Hematocrit levels are elevated in the first 24 hours after injury as a result of hemoconcentration from the loss of intravascular fluid. The normal hematocrit level ranges from 37% to 47% (0.37 to 0.47) (female) and 42% to 52% (0.42 to 0.52) (male). The remaining options identify normal laboratory values.

The nurse is caring for a client immediately after removal of the endotracheal tube. The nurse should report which sign immediately if experienced by the client? 1. Stridor 2. Occasional pink-tinged sputum 3. Respiratory rate of 24 breaths/minute 4.A few basilar lung crackles on the right

Rationale: 1 Following removal of the endotracheal tube the nurse monitors the client for respiratory distress. The nurse reports stridor to the health care provider (HCP) immediately. This is a high-pitched, coarse sound that is heard with the stethoscope over the trachea. Stridor indicates airway edema and places the client at risk for airway obstruction. Although the findings identified in the remaining options require monitoring, they do not require immediate notification of the HCP.

The nurse is caring for a client with a chest tube drainage system. During repositioning of the client, the chest tube accidentally pulls out of the pleural cavity. Which is the initial nursing action? 1. Apply an occlusive dressing. 2. Reinsert the chest tube quickly. 3. Contact the respiratory therapist. 4. Contact the health care provider (HCP).

Rationale: 1 If a chest tube is accidentally pulled out, the nurse would immediately apply an occlusive dressing and then contact the HCP. It is not appropriate and not a nursing role to reinsert a chest tube. It is not necessary to contact the respiratory therapist. The HCP needs to be notified, but this is not the initial nursing action.

The nurse is providing morning care to a client who has a closed chest tube drainage system to treat a pneumothorax. When the nurse turns the client to the side, the chest tube is accidentally dislodged from the chest. The nurse immediately applies sterile gauze over the chest tube insertion site. Which is the nurse's next action? 1. Call the health care provider. 2. Replace the chest tube system. 3. Obtain a pulse oximetry reading. 4. Place the client in a Trendelenburg's position.

Rationale: 1 If the chest drainage system is dislodged from the insertion site, the nurse immediately applies sterile gauze over the site and calls the health care provider. The nurse would maintain the client in an upright position. A new chest tube system may be attached if the tube requires insertion, but this would not be the next action. Pulse oximetry readings would assist in determining the client's respiratory status, but the priority action would be to call the health care provider in this emergency situation.

The nurse is caring for a client with a chest tube who accidentally disconnects the tube from the drainage system when trying to get out of bed. The nurse immerses the end of the tube in sterile water. What immediate action should the nurse take? 1. Obtain a new drainage system. 2. Ask the client to hold his or her breath. 3. Place the client in a prone position. 4. Place a sterile dressing over the chest tube insertion site.

Rationale: 1 If the drainage system is broken or interrupted or the tube disconnects, the end of the tube should be placed in a bottle of sterile water held below the level of the chest. A new drainage system is then immediately obtained and set up. Placing the client in the prone position and asking the client to hold his or her breath are not helpful. Placing a sterile dressing over the disconnection site will not prevent complications resulting from the tube disconnection. The nurse should also perform an assessment on the client and contact the health care provider.

A client begins to experience drainage of small amounts of bright red blood from the tracheostomy tube 24 hours after a supraglottic laryngectomy. Which is the best nursing action? 1. Notify the health care provider (HCP). 2. Increase the frequency of suctioning. 3. Add moisture to the oxygen delivery system. 4. Document the character and amount of drainage.

Rationale: 1 Immediately after laryngectomy, a small amount of bleeding occurs from the tracheostomy that resolves within the first few hours. Otherwise, bleeding that is bright red may be a sign of impending rupture of a vessel. The bleeding in this instance represents a potential threat to life, and the HCP is notified to further evaluate the client and suture or repair the source of the bleeding. The other options do not address the urgency of the problem. Failure to notify the HCP places the client at risk.

The industrial nurse is providing instructions to a group of employees regarding care to a client in the event of a chemical burn injury. The nurse instructs the employees that which is the first consideration in immediate care? 1. Removing all clothing, including gloves, shoes, and any undergarments 2. Determining the antidote for the chemical and placing the antidote on the burn site 3. Leaving all clothing in place until the client is brought to the emergency department 4. Lavaging the skin with water and avoiding brushing powdered chemicals off the clothing

Rationale: 1 In a chemical burn injury the burning process continues as long as the chemical is in contact with the skin. All clothing, including gloves, shoes, and undergarments, is removed immediately, and water lavage is instituted before and during transport to the emergency department. Powdered chemicals are first brushed off the client before lavage is performed.

The nurse is developing a nursing care plan for a client with a circumferential burn injury of the right arm. What is the nurse's priority action? 1. Monitor the radial pulse every hour. 2. Keep the extremity in a dependent position. 3. Document any changes that occur in the pulse. 4. Place pressure dressings and wraps around the burn sites.

Rationale: 1 In a client with ineffective tissue perfusion related to a circumferential burn injury, peripheral pulses should be assessed every hour for 72 hours. The affected extremities should be elevated, and the health care provider should be notified of any changes in pulses, capillary refill, or pain sensation. Pressure dressings and wraps should not be applied around the circumferential burn because they could cause a further alteration in peripheral circulation.

A woman was working in her garden. She accidentally sprayed insecticide into her right eye. She calls the emergency department, frantic and screaming for help. The nurse should instruct the woman to take which immediate action? 1. Irrigate the eyes with water. 2. Come to the emergency department. 3. Call the health care provider (HCP). 4. Irrigate the eyes with diluted hydrogen peroxide.

Rationale: 1 In this type of accident, the client is instructed to irrigate the eyes immediately with running water for at least 20 minutes, or until the emergency medical services personnel arrive. In the emergency department, the cleansing agent of choice is usually normal saline. Calling the HCP and going to the emergency department delays necessary intervention. Hydrogen peroxide is never placed in the eyes.

A client is admitted to the hospital with a diagnosis of neurogenic shock after a traumatic motor vehicle collision. Which manifestation best characterizes this diagnosis? 1. Bradycardia 2. Hyperthermia 3. Hypoglycemia 4. Increased cardiac output

Rationale: 1 Neurogenic shock can occur after a spinal cord injury. Usually the body attempts to compensate massive vasodilation by becoming tachycardic to increase the amount of blood flow and oxygen delivered to the tissues; however, in neurogenic shock, the sympathetic nervous system is disrupted, so the parasympathetic system takes over, resulting in bradycardia. This insufficient pumping of the heart leads to a decrease in cardiac output. Hypoglycemia is not an indicator of neurogenic shock. Hypothermia develops because of the vasodilation and the inability to control body temperature through vasoconstriction.

The nurse is developing a plan of care for a client who sustained an inhalation burn injury. Which nursing intervention should the nurse include in the plan of care for this client? 1. Elevate the head of the bed. 2. Monitor oxygen saturation levels every 4 hours. 3. Encourage coughing and deep breathing every 4 hours. 4. Assess respiratory rate and breath sounds every 4 hours.

Rationale: 1 Nursing interventions for the client with an inhalation burn injury include assessing the respiratory rate every hour, monitoring oxygen saturation levels every hour, and assisting the client in coughing and deep breathing every hour. The head of the bed is elevated to facilitate lung expansion.

The nurse is planning care for a client being admitted to the nursing unit who attempted suicide. Which priority nursing intervention should the nurse include in the plan of care? 1. One-to-one suicide precautions 2. Suicide precautions with 30-minute checks 3. Checking the whereabouts of the client every 15 minutes 4. Asking the client to report suicidal thoughts immediately

Rationale: 1 One-to-one suicide precautions are required for a client who has attempted suicide. Options 2 and 3 may be appropriate, but not at the present time, considering the situation. Option 4 also may be an appropriate nursing intervention, but the priority is identified in the correct option. The best intervention is constant supervision so that the nurse may intervene as needed if the client attempts to harm himself or herself.

A client experienced an open pneumothorax (sucking wound), which has been covered with an occlusive dressing. The client begins to experience severe dyspnea, and the blood pressure begins to fall. The nurse should first perform which action? 1. Remove the dressing. 2. Reinforce the dressing. 3. Call the health care provider (HCP). 4. Measure oxygen saturation by oximetry.

Rationale: 1 Placement of a dressing over a chest wound could convert an open pneumothorax to a closed (tension) pneumothorax. This may result in a sudden decline in respiratory status, mediastinal shift with twisting of the great vessels, and circulatory compromise. If clinical changes occur, the nurse should remove the dressing immediately, allowing air to escape. Therefore, reinforcing the dressing is an incorrect action. The nurse would measure oxygen saturation by oximetry and would call the HCP, but these would not be the first actions in this situation.

A client has been admitted with chest trauma after a motor vehicle crash and has undergone subsequent intubation. The nurse checks the client when the high-pressure alarm on the ventilator sounds, and notes that the client has absence of breath sounds in the right upper lobe of the lung. The nurse immediately assesses for other signs of which condition? 1. Right pneumothorax 2. Pulmonary embolism 3. Displaced endotracheal tube 4. Acute respiratory distress syndrome

Rationale: 1 Pneumothorax is characterized by restlessness, tachycardia, dyspnea, pain with respiration, asymmetrical chest expansion, and diminished or absent breath sounds on the affected side. Pneumothorax can cause increased airway pressure because of resistance to lung inflation. Acute respiratory distress syndrome and pulmonary embolism are not characterized by absent breath sounds. An endotracheal tube that is inserted too far can cause absent breath sounds, but the lack of breath sounds most likely would be on the left side because of the degree of curvature of the right and left mainstem bronchi.

The community health nurse is providing a teaching session to firefighters in a small community regarding care of a burn victim at the scene of injury. The nurse instructs the firefighters that in the event of a tar burn, which is the immediate action? 1. Cooling the injury with water 2. Removing all clothing immediately 3. Removing the tar from the burn injury 4. Leaving any clothing that is saturated with tar in place

Rationale: 1 Scald burns and tar or asphalt burns are treated by immediate cooling with saline solution or water, if available, or immediate removal of the saturated clothing. Clothing that is burned into the skin is not removed because increased tissue damage and bleeding may result. No attempt is made to remove tar from the skin at the scene.

During the early postoperative period, a client who has undergone a cataract extraction complains of nausea and severe eye pain over the operative site. What should be the initial nursing action? 1. Call the health care provider (HCP). 2. Reassure the client that this is normal. 3. Turn the client onto his or her operative side. 4. Administer the prescribed pain medication and antiemetic.

Rationale: 1 Severe pain or pain accompanied by nausea following a cataract extraction is an indicator of increased intraocular pressure and should be reported to the HCP immediately. Options 2, 3, and 4 are inappropriate actions.

The nurse is monitoring a client who required a Sengstaken-Blakemore tube because other measures for treating bleeding esophageal varices were unsuccessful. The client complains of severe pain of abrupt onset. Which nursing action is most appropriate? 1. Cut the tube. 2. Reposition the client. 3. Assess the lumens of the tubes. 4. Administer the prescribed analgesics.

Rationale: 1 Spontaneous rupture of the gastric balloon, upward migration of the tube, and occlusion of the airway are possible complications associated with a Sengstaken-Blakemore tube. Esophageal rupture also may occur and is characterized by the abrupt onset of severe pain. In the event of any of these life-threatening emergencies, the tube is cut and removed.

A client with depression receiving phenelzine sulfate suddenly complains of a severe headache and neck stiffness and soreness and then begins to vomit. The nurse takes the client's blood pressure and notes that it is 210/102 mm Hg. On the basis of the findings, the nurse should obtain which medication from the emergency drawer of the medication cart? 1. Phentolamine 2. Protamine sulfate 3. Calcium gluconate 4. Phenobarbital sodium

Rationale: 1 The antidote for hypertensive crisis is phentolamine. Hypertensive crisis may be manifested by hypertension, occipital headache radiating frontally, neck stiffness and soreness, nausea, vomiting, sweating, fever and chills, clammy skin, dilated pupils, and palpitations. Tachycardia or bradycardia and constricting chest pain also may be present.

A client had a 1000-mL bag of 5% dextrose in 0.9% sodium chloride hung at 1500. The nurse, making rounds at 1545, finds that the client is apprehensive, complaining of a pounding headache, is dyspneic with chills, and has an increased pulse rate. The intravenous (IV) bag has 400 mL remaining. The nurse should take which action first? 1. Shut off the infusion. 2. Sit the client up in bed. 3. Remove the angiocatheter and IV quickly. 4. Place the client in the Trendelenburg's position.

Rationale: 1 The client's symptoms are compatible with speed shock (fluid volume overload). This may be verified by noting that 600 mL has infused in the course of 45 minutes. The first action of the nurse from the options presented is to shut off the infusion. The nurse may elevate the head of the bed to aid the client's breathing. Placing the client in the Trendelenburg's position is not an appropriate action and will increase the dyspnea. The nurse also notifies the health care provider. The angiocatheter is not removed. An IV access may be needed to treat the complication and may continue to be needed when the complication has been managed.

The nurse reviewing the operative record for a client who has just undergone cardiac surgery notes that the client's cardiac output immediately after surgery was 3.6 L/min. Which intervention is appropriate based on the client's cardiac output reading? 1. Notify the health care provider (HCP). 2. Continue to monitor the cardiac output. 3. Place the client in the shock position. 4. Increase the intravenous (IV) fluid rate.

Rationale: 1 The normal cardiac output for the adult can range from 4 to 8 L/min and varies greatly with body size. The heart normally pumps 5 L of blood every minute. The HCP should be notified because the result falls below the normal limit. Continuing to monitor the client dismisses the fact that the cardiac output is decreased. Increasing fluids may exacerbate the cardiac workload, and the nurse should consult with the HCP before this intervention. There are no data to suggest the client is currently in shock, so placing the client in shock position would be of no benefit.

A client presents to the urgent care center with complaints of abdominal pain and vomits bright red blood. Which is the priority nursing action? 1. Take the client's vital signs. 2. Perform a complete abdominal assessment. 3. Obtain a thorough history of the recent health status. 4. Prepare to insert a nasogastric tube and test pH and occult blood.

Rationale: 1 The nurse should take the client's vital signs first to determine whether they are hypovolemic or in shock from blood loss. This also provides a baseline blood pressure and pulse by which to gauge the effectiveness of treatment. Signs and symptoms of shock include low blood pressure; rapid, weak pulse; increased thirst; cold, clammy skin; and restlessness. Therefore, the remaining options are incorrect.

A client in cardiogenic shock had an intra-aortic balloon pump inserted 24 hours earlier via the left femoral approach. The nurse notes that the client's left foot is cool and mottled and the left pedal pulse is weak. Which action should the nurse take? 1. Call the health care provider immediately. 2. Document these findings, which are expected. 3. Re-evaluate the neurovascular status in 1 hour. 4. Increase the rate of the intravenous nitroglycerin infusion.

Rationale: 1 The nursing interventions for the client with an intra-aortic balloon pump are the same as for any client who has had cardiovascular surgery. The peripheral circulation to the affected limb is monitored for signs of occlusion, such as coolness, mottling, pain, tingling, and decreased or absent distal pulse. Adverse changes are reported immediately. The remaining options are incorrect.

The client sustains a contusion of the eyeball following a traumatic injury with a blunt object. Which intervention should be initiated immediately? 1. Apply ice to the affected eye. 2. Irrigate the eye with cool water. 3. Notify the health care provider (HCP). 4. Accompany the client to the emergency department.

Rationale: 1 Treatment for a contusion begins at the time of injury. Ice is applied immediately. The client then should be seen by an HCP and receive a thorough eye examination to rule out the presence of other eye injuries.

The client who has experienced a myocardial infarction (MI) is recovering from cardiogenic shock. The nurse knows that which observation of the client's clinical condition is most favorable? 1. Urine output of 40 mL/hr 2. Heart rate of 110 beats/minute 3. Frequent premature ventricular contractions 4. Central venous pressure (CVP) of 15 mm Hg

Rationale: 1 Urine output of greater than 30 mL/hr indicates adequate perfusion to the kidneys, so the other organs are most likely equally perfused. Classic cardiovascular signs of cardiogenic shock include low blood pressure and tachycardia. Dysrhythmias commonly occur as a result of decreased oxygenation to the myocardium and are not a favorable sign. The CVP rises as the effects of the backward blood flow caused by the left ventricular failure became apparent.

A client has frequent bursts of ventricular tachycardia on the cardiac monitor. What should the nurse be most concerned about with this dysrhythmia? 1. It can develop into ventricular fibrillation at any time. 2. It is almost impossible to convert to a normal rhythm. 3. It is uncomfortable for the client, giving a sense of impending doom. 4. It produces a high cardiac output that quickly leads to cerebral and myocardial ischemia.

Rationale: 1 Ventricular tachycardia is a life-threatening dysrhythmia that results from an irritable ectopic focus that takes over as the pacemaker for the heart. The low cardiac output that results can lead quickly to cerebral and myocardial ischemia. Clients frequently experience a feeling of impending doom. Ventricular tachycardia is treated with antidysrhythmic medications, cardioversion (if the client is awake), or defibrillation (loss of consciousness). Ventricular tachycardia can deteriorate into ventricular fibrillation at any time.

A client develops an anaphylactic reaction after receiving morphine. The nurse should plan to institute which actions? Select all that apply. 1. Administer oxygen. 2. Quickly assess the client's respiratory status. 3. Document the event, interventions, and client's response. 4. Leave the client briefly to contact a health care provider (HCP). 5. Keep the client supine regardless of the blood pressure readings. 6. Start an intravenous (IV) infusion of D5W and administer a 500-mL bolus.

Rationale: 1,2,3 An anaphylactic reaction requires immediate action, starting with quickly assessing the client's respiratory status. Although the HCP and the Rapid Response Team must be notified immediately, the nurse must stay with the client. Oxygen is administered and an IV of normal saline is started and infused per HCP prescription. Documentation of the event, actions taken, and client outcomes needs to be done. The head of the bed should be elevated if the client's blood pressure is normal.

Which interventions would be included in the care of a client with a head injury and a subarachnoid bolt? Select all that apply. 1. Monitor vital signs. 2. Monitor neurological status. 3. Monitor the dressing for signs of infection. 4. Monitor for signs of increased intracranial pressure. 5. Drain cerebrospinal fluid when the intracranial pressure is elevated.

Rationale: 1,2,3,4 A subarachnoid bolt is inserted into the subarachnoid space and is used to measure intracranial pressure. Because a subarachnoid bolt is placed in the subarachnoid space, it is not capable of draining cerebrospinal fluid, which is produced in the ventricles. Therefore, the option to drain cerebrospinal fluid is not an intervention. The remaining options are appropriate interventions.

The client who has had radical neck dissection begins to hemorrhage at the incision site. Which immediate actions should the nurse take? Select all that apply. 1. Monitor the client's airway. 2. Call the Rapid Response Team. 3. Call the health care provider (HCP). 4. Apply manual pressure over the site. 5. Lower the head of the bed to a flat position.

Rationale: 1,2,3,4 If the client begins to hemorrhage from the surgical site following radical neck dissection, the nurse elevates the head of the bed to maintain airway patency and prevent aspiration. The nurse applies pressure over the bleeding site and stays with the client and ask someone to call the Rapid Response Team and the HCP immediately.

The nurse in a medical unit is caring for a client with heart failure. The client suddenly develops extreme dyspnea, tachycardia, and lung crackles and the nurse suspects pulmonary edema. The nurse immediately asks another nurse to contact the health care provider and prepares to implement which priority interventions? Select all that apply. 1. Administering oxygen 2. Inserting a Foley catheter 3. Administering furosemide 4. Administering morphine sulfate intravenously 5. Transporting the client to the coronary care unit 6. Placing the client in a low Fowler's side-lying position

Rationale: 1,2,3,4 Pulmonary edema is a life-threatening event that can result from severe heart failure. In pulmonary edema, the left ventricle fails to eject sufficient blood, and pressure increases in the lungs because of the accumulated blood. Oxygen is always prescribed, and the client is placed in a high Fowler's position to ease the work of breathing. Furosemide, a rapid-acting diuretic, will eliminate accumulated fluid. A Foley catheter is inserted to measure output accurately. Intravenously administered morphine sulfate reduces venous return (preload), decreases anxiety, and also reduces the work of breathing. Transporting the client to the coronary care unit is not a priority intervention. In fact, this may not be necessary at all if the client's response to treatment is successful.

A client with type 1 diabetes mellitus in the emergency department is diagnosed with diabetic ketoacidosis (DKA). Which interventions should the nurse anticipate being prescribed initially? Select all that apply. 1. Monitoring urine for ketones 2. Intravenous potassium replacement 3. Administration of intravenous insulin 4. A bolus of 5% dextrose intravenously 5. Administration of a liter of 0.9% NaCl intravenously.

Rationale: 1,2,3,5 DKA is caused by a profound deficiency of insulin and is characterized by hyperglycemia (blood glucose level greater than or equal to 250 mg/dL [13.9 mmol/L]), ketosis (ketones in urine or serum), metabolic acidosis, dehydration, and hypokalemia. Therefore, the correct options are 1, 2, 3, and 5. Option 4 is incorrect, as clients with DKA have a profound deficiency of insulin and an elevated blood glucose; therefore, dextrose would not be administered as an initial intervention.

A client has had radical neck dissection and begins to hemorrhage at the incision site. The nurse should take which actions in this situation? Select all that apply. 1. Monitor vital signs. 2. Monitor the client's airway. 3. Apply manual pressure over the site. 4. Lower the head of the bed to a flat position. 5. Call the health care provider (HCP) immediately.

Rationale: 1,2,3,5 If the client begins to hemorrhage from the surgical site after radical neck dissection, the nurse elevates the head of the bed to maintain airway patency and prevent aspiration. The nurse applies pressure over the bleeding site and calls the HCP immediately. The nurse also monitors the client's airway and vital signs.

The nurse is providing care for a client with new onset of a dysrhythmia. The nurse anticipates which prescriptions from the health care provider? Select all that apply. Refer to Figure. 1. Oxygen therapy 2. An echocardiogram 3. An intravenous dose of metoprolol 4. One dose of atropine to promote slowing of the rate 5. A bolus of intravenous heparin followed by a continuous infusion

Rationale: 1,2,3,5 In atrial fibrillation the ventricles often beat with a rapid rate in response to the numerous atrial impulses. Heart dilation and blood pooling in the atria can lead to thrombus formation, which increases the risk for stroke or other embolic events; therefore, heparin is indicated. The rapid and irregular ventricular rate decreases ventricular filling and reduces cardiac output, further impairing the heart's perfusion ability. Therefore, oxygen and metoprolol (to slow the ventricular response) are appropriate. An echocardiogram will help to assess heart valve function because mitral valve disease can lead to atrial fibrillation. Atropine will increase the heart rate further.

The occupational health nurse is called to care for an employee who experienced a traumatic amputation of a finger. Which actions should the nurse take to provide emergency care and prepare the client for transport to the hospital? Select all that apply. 1. Elevate the extremity above heart level. 2. Assess the employee for airway or breathing problems. 3. Remove the layered gauze every 10 minutes to check the bleeding. 4. Wrap the severed finger in moistened gauze, and place it in a bag of ice water. 5. Examine the amputation site and apply direct pressure to the site using layers of gauze.

Rationale: 1,2,5 The nurse would assess the victim for airway or breathing problems immediately. Then the nurse would examine the amputation site and apply direct pressure using layers of dry gauze or another type of cloth. Sterile gloves and sterile gauze should always be used if available. If sterile materials are not available, clean materials should be used if possible. The gauze that is applied is a pressure dressing and is not removed because of the risk of dislodgment of a clot that may be forming; the pressure dressing will be removed at the hospital. The extremity is elevated above the victim's heart level to decrease the bleeding. The severed finger should be wrapped in dry, sterile gauze (if available) or a clean cloth. It is placed in a watertight, sealed plastic bag. Then the watertight, sealed plastic bag is placed in a bag of ice water. The severed finger is never placed directly on ice; contact between the finger and water is avoided because of the risk of tissue damage. The severed part is transported to the hospital with the victim for possible replanting. Additionally, emergency medical services is called to transport the victim to the hospital.

A client being hemodialyzed suddenly becomes short of breath and complains of chest pain. The client is tachycardic, pale, and anxious and the nurse suspects air embolism. What are the priority nursing actions? Select all that apply. 1. Administer oxygen to the client. 2. Continue dialysis at a slower rate after checking the lines for air. 3. Notify the health care provider (HCP) and Rapid Response Team. 4. Stop dialysis, and turn the client on the left side with head lower than feet. 5. Bolus the client with 500 mL of normal saline to break up the air embolus.

Rationale: 1,3,4 If the client experiences air embolus during hemodialysis, the nurse should terminate dialysis immediately, position the client so the air embolus is in the right side of the heart, notify the HCP and Rapid Response Team, and administer oxygen as needed. Slowing the dialysis treatment or giving an intravenous bolus will not correct the air embolism or prevent complications.

A client is brought to the emergency department complaining of substernal chest pain. To distinguish between angina and myocardial infarction, the nurse assesses for which characteristics of angina? Select all that apply. 1. Chest pain that resolves with rest 2. Chest pain requiring an opioid for relief 3. Chest pain that is relieved by nitroglycerin 4. Chest pain that lasts longer than 30 minutes 5. Chest pain that is usually precipitated by exertion

Rationale: 1,3,5 Angina is chest pain caused by a temporary imbalance between the coronary arteries' ability to supply oxygen and the cardiac muscle's demand for oxygen. Myocardial infarction refers to injury and necrosis of myocardial tissue that occurs when the tissue is abruptly and severely deprived of oxygen. When a client complains of chest pain, it is critical that treatment is immediately initiated and that the nurse assesses for characteristics of angina versus those associated with myocardial infarction. Angina is characterized by substernal chest pain radiating to the left arm. The pain is usually precipitated by exertion or stress, is relieved by nitroglycerin or rest, and lasts less than 15 minutes. Characteristics of myocardial infarction include substernal chest pain that radiates to the left arm; pain in the jaw, abdomen, back, or shoulder can also occur. The substernal chest pain occurs without cause, usually in the morning; is relieved only by opioids, and lasts 30 minutes or longer.

An emergency department nurse is caring for a child with suspected acute epiglottitis. Which nursing interventions apply in the care of this child? Select all that apply. 1. Ensure a patent airway. 2. Obtain a throat culture. 3. Maintain the child in a supine position. 4. Obtain a pediatric-size tracheostomy tray. 5. Prepare the child for a chest radiographic study. 6. Place the child on an oxygen saturation monitor.

Rationale: 1,4,5,6 Acute epiglottitis is a serious obstructive inflammatory process that requires immediate intervention. The nurse immediately ensures a patent airway. To reduce respiratory distress, the child should sit upright. Examining the throat with a tongue depressor or attempting to obtain a throat culture is contraindicated because it could precipitate further obstruction. A complete blood count is obtained, and the child is placed on an oxygen saturation monitor. Lateral neck and chest radiographic films are obtained to determine the degree of obstruction, if present. A pediatric-size tracheostomy tray should be readily available, and intubation may be necessary if respiratory distress is severe.

A new nursing graduate is caring for a client who is attached to a cardiac monitor. While assisting the client with bathing, the nurse observes the sudden development of ventricular tachycardia (VT), but the client remains alert and oriented and has a pulse. Which interventions would the nurse take? Select all that apply. 1. Administer oxygen. 2. Defibrillate the client. 3. Obtain an electrocardiogram (ECG). 4. Contact the health care provider (HCP). 5. Assess circulation, airway, and breathing. 6. Initiate cardiopulmonary resuscitation (CPR).

Rationale: 1,4,5,6 With VT in a stable client, the nurse assesses circulation, airway, and breathing; administers oxygen; and confirms the rhythm via a 12-lead ECG. The HCP is contacted, and antidysrhythmics may be prescribed. With pulseless VT, the HCP or a specially trained nurse must immediately defibrillate the client or initiate CPR followed by defibrillation as soon as possible.

When creating a mechanically ventilated client's plan of care for prevention of ventilator-associated pneumonia (VAP), the nurse should include which measures in the plan? Select all that apply. 1. Suction the oral cavity whenever needed. 2. Apply topical antibiotics to the oral cavity. 3. Change the ventilator circuit tubing every 2 hours. 4. Maintain the client in a supine position at all times. 5. Practice frequent oral hygiene, including teeth brushing. 6. Wear gloves when suctioning or handling the endotracheal tube.

Rationale: 1,5,6 Measures to prevent VAP include removing subglottic secretions every 2 hours and whenever needed; frequent oral hygiene, which includes teeth brushing; hand hygiene before and after each client contact; and application of gloves after hand washing and before suctioning. Topical antibiotics have no effect. Ventilator circuit tubing does not need to be changed every 2 hours; depending on agency policy, tubing is changed every 24 or 48 hours. Head of the bed elevation is maintained at a minimum of 30 degrees; a supine position can lead to aspiration.

A 5-year-old boy is brought by his mother to the emergency department after ingesting a bottle of acetylsalicylic acid. Which procedure should be initially instituted with this child? 1. Administer ipecac by mouth and monitor emesis. 2. Institute a gastric lavage and administer activated charcoal. 3. Administer a chelating agent such as edetate calcium disodium. 4. Institute a gastric lavage and administer the antidote acetylcysteine.

Rationale: 2 A gastric lavage must be performed after ingestion of acetylsalicylic acid, and activated charcoal is administered to prevent further absorption of the substance. N-acetylcysteine is the antidote for acetaminophen. Administering ipecac or edetate calcium disodium is not a treatment measure for acetylsalicylic acid poisoning. Edetate calcium disodium may be prescribed for the treatment of lead poisoning. Ipecac causes vomiting, and this substance is used only in specific poisoning conditions; in this situation, vomiting can cause irritation of the esophagus.

A client arrives in the emergency department following an automobile crash. The client's forehead hit the steering wheel and a hyphema is diagnosed. The nurse should place the client in which position? 1. Flat in bed 2. A semi Fowler's position 3. Lateral on the affected side 4. Lateral on the unaffected side

Rationale: 2 A hyphema is the presence of blood in the anterior chamber. Hyphema is produced when a force is sufficient to break the integrity of the blood vessels in the eye and can be caused by direct injury, such as a penetrating injury from a BB or pellet, or indirectly, such as from striking the forehead on a steering wheel during an accident. The client is treated by bed rest in a semi Fowler's position to assist gravity in keeping the hyphema away from the optical center of the cornea.

The nurse has a prescription to administer acetylcysteine to a client admitted to the emergency department with acetaminophen overdose. Before giving this medication, what is the nurse's best action? 1. Administer the full-strength solution. 2. Empty the stomach by emesis or lavage. 3. Check that the antidote is readily available. 4. Ensure that the client knows how to use a nebulizer.

Rationale: 2 Acetylcysteine can be given orally or by nasogastric tube to treat acetaminophen overdose, or it may be given by inhalation for use as a mucolytic. Before giving the medication as an antidote to acetaminophen, the nurse ensures that the client's stomach is empty through emesis or gastric lavage. The solution is diluted in cola, water, or juice to make it more palatable. It is then administered orally or by nasogastric tube. Acetylcysteine is the antidote to acetaminophen.

A client whose cardiac rhythm was normal sinus rhythm suddenly exhibits a different rhythm on the monitor. The nurse should take which action? Refer to Figure. 1. Continue to watch the monitor. 2. Contact the health care provider (HCP). 3. Check to see if cardiac medications are due. 4. Call respiratory therapy for a respiratory treatment.

Rationale: 2 Atrial fibrillation is characterized by multiple rapid impulses from many atrial foci in a totally disorganized manner at a rate of 350 to 600 times per minute. The atria quiver in fibrillation. As a result, thrombi can form within the right atrium and move through the right ventricle to the lungs. This can be a life-threatening situation requiring pharmacological therapy. Therefore, the HCP needs to be contacted. Continuing to monitor and checking to see if cardiac medications are due delay necessary and required interventions. A respiratory treatment is not useful for this situation.

A client with myocardial infarction is developing cardiogenic shock. Because of the risk of myocardial ischemia, what condition should the nurse carefully assess the client for? 1. Bradycardia 2. Ventricular dysrhythmias 3. Rising diastolic blood pressure 4. Falling central venous pressure

Rationale: 2 Classic signs of cardiogenic shock as they relate to myocardial ischemia include low blood pressure and tachycardia. The central venous pressure would rise as the backward effects of the severe left ventricular failure became apparent. Dysrhythmias commonly occur as a result of decreased oxygenation and severe damage to greater than 40% of the myocardium.

The nurse has developed a nursing care plan for a client with a burn injury to implement during the emergent phase. Which priority intervention should the nurse include in the plan of care? 1. Monitor vital signs every 4 hours. 2. Monitor mental status every hour. 3. Monitor intake and output every shift. 4. Obtain and record weight every other day.

Rationale: 2 During the emergent phase after a burn injury, because of fluid volume deficits secondary to a burn injury, vital signs should be monitored every hour (every 4 hours is too infrequent) until the client is hemodynamically stable. The nurse should monitor the mental status of the client every hour for the first 48 hours. The weight should be obtained and recorded daily or twice daily, and intake and output measurements should be recorded on an hourly basis.

The nurse is caring for a client who had a resection of an abdominal aortic aneurysm yesterday. The client has an intravenous (IV) infusion at a rate of 150 mL/hour, unchanged for the last 10 hours. The client's urine output for the last 3 hours has been 90, 50, and 28 mL (28 mL is most recent). The client's blood urea nitrogen level is 35 mg/dL (12.6 mmol/L) and the serum creatinine level is 1.8 mg/dL (159 mcmol/L), measured this morning. Which nursing action is the priority? 1. Check the urine specific gravity. 2. Call the health care provider (HCP). 3. Put the IV line on a pump so that the infusion rate is sure to stay stable. 4. Check to see if the client had a blood sample for a serum albumin level drawn.

Rationale: 2 Following abdominal aortic aneurysm resection or repair, the nurse monitors the client for signs of acute kidney injury. Acute kidney injury can occur because often much blood is lost during the surgery and, depending on the aneurysm location, the renal arteries may be hypoperfused for a short period during surgery. Normal reference levels are BUN, 10-20 mg/dL (3.6-7.1 mmol/L), and creatinine, male, 0.6-1.2 mg/dL (53-106 mcmol/L) and female, 0.5-1.1 mg/dL (44-97 mcmol/L). Options 1 and 4 are not associated with the data in the question. The IV should have already been on a pump. Urine output lower than 30 mL/hour is reported to the HCP.

The nurse caring for a client with a chest tube turns the client to the side and the chest tube accidentally disconnects from the water seal chamber. Which initial action should the nurse take? 1. Call the health care provider (HCP). 2. Place the tube in a bottle of sterile water. 3. Replace the chest tube system immediately. 4. Place a sterile dressing over the disconnection site.

Rationale: 2 If the chest drainage system is disconnected, the end of the tube is placed in a bottle of sterile water held below the level of the chest. The HCP may need to be notified, but this is not the initial action. The system is replaced if it breaks or cracks or if the collection chamber is full. Placing a sterile dressing over the disconnection site will not prevent complications resulting from the disconnection.

A client arrives in the emergency department with a penetrating eye injury from wood chips that occurred while cutting wood. The nurse assesses the eye and notes a piece of wood protruding from the eye. What is the initial nursing action? 1. Apply an eye patch. 2. Perform visual acuity tests. 3. Irrigate the eye with sterile saline. 4. Remove the piece of wood using a sterile eye clamp.

Rationale: 2 If the eye injury is the result of a penetrating object, the object may be noted protruding from the eye. This object must never be removed except by the ophthalmologist because it may be holding ocular structures in place. Application of an eye patch or irrigation of the eye may disrupt the foreign body and cause further tearing of the cornea.

While changing the tapes on a newly inserted tracheostomy tube, the client coughs and the tube is dislodged. Which is the initial nursing action? 1. Call the health care provider to reinsert the tube. 2. Grasp the retention sutures to spread the opening. 3. Call the respiratory therapy department to reinsert the tracheotomy. 4. Cover the tracheostomy site with a sterile dressing to prevent infection.

Rationale: 2 If the tube is dislodged accidentally, the initial nursing action is to grasp the retention sutures and spread the opening. If agency policy permits, the nurse then attempts to replace the tube immediately. Calling ancillary services or the health care provider will delay treatment in this emergency situation. Covering the tracheostomy site will block the airway.

A child undergoes surgical removal of a brain tumor. During the postoperative period, the nurse notes that the child is restless, the pulse rate is elevated, and the blood pressure has decreased significantly from the baseline value. The nurse suspects that the child is in shock. Which is the most appropriate nursing action? 1. Place the child in a supine position. 2. Notify the health care provider (HCP). 3. Place the child in Trendelenburg's position. 4. Increase the flow rate of the intravenous fluids.

Rationale: 2 In the event of shock, the HCP is notified immediately before the nurse changes the child's position or increases intravenous fluids. After craniotomy, a child is never placed in the supine or Trendelenburg's position because it increases intracranial pressure (ICP) and the risk of bleeding. The head of the bed should be elevated. Increasing intravenous fluids can cause an increase in ICP.

A client experiencing cocaine toxicity is brought to the emergency department. The nurse should prepare to take which initial action? 1. Administer naloxone. 2. Ensure a patent airway. 3. Establish an intravenous access. 4. Obtain a 12-lead electrocardiogram (ECG).

Rationale: 2 Initial management when caring for a client with cocaine toxicity is to ensure a patent airway. Although options 1, 3, and 4 are components of care, airway is the priority.

An emergency department nurse is caring for a conscious child who was brought to the emergency department after the ingestion of half a bottle of acetylsalicylic acid (aspirin). The nurse anticipates that which will be the initial treatment? 1. Placement of a dialysis catheter 2. The administration of an emetic 3. The administration of vitamin K 4. The administration of sodium bicarbonate

Rationale: 2 Initial treatment of acetylsalicylic acid overdose includes the administration of an emetic or gastric lavage. Activated charcoal may be administered to decrease absorption. Fluids and sodium bicarbonate may be administered intravenously to enhance excretion but would not be the initial treatment. Dialysis is used in extreme cases if the child is unresponsive to therapy. Vitamin K is the antidote for warfarin sodium overdose.

The nurse is monitoring a child who is receiving ethylenediaminetetraacetic acid (EDTA) with BAL (British anti-Lewisite) for the treatment of lead poisoning. The nurse reviews the laboratory results for the child during treatment with this medication and is particularly concerned with monitoring which laboratory test result? 1. Cholesterol level 2. Blood urea nitrogen (BUN) level 3. Complete blood cell (CBC) count 4. Hemoglobin and hematocrit (H&H) levels

Rationale: 2 Kidney function tests should be monitored because EDTA is nephrotoxic. The calcium level should also be monitored because EDTA enhances the excretion of calcium. The remaining laboratory results are not the primary concern during treatment with EDTA.

A client with myocardial infarction suddenly becomes tachycardic, shows signs of air hunger, and begins coughing frothy, pink-tinged sputum. Which finding would the nurse anticipate when auscultating the client's breath sounds? 1. Stridor 2. Crackles 3. Scattered rhonchi 4. Diminished breath sounds

Rationale: 2 Pulmonary edema is characterized by extreme breathlessness, dyspnea, air hunger, and the production of frothy, pink-tinged sputum. Auscultation of the lungs reveals crackles. Rhonchi and diminished breath sounds are not associated with pulmonary edema. Stridor is a crowing sound associated with laryngospasm or edema of the upper airway.

A client begins experiencing wheezing, anxiety, swelling, and hives after eating shellfish and is brought to the emergency department. Which immediate action should the nurse implement? 1. Administer epinephrine. 2. Maintain a patent airway. 3. Administer a corticosteroid. 4. Apply a MedicAlert bracelet.

Rationale: 2 Swelling, hives, lowered blood pressure, anxiety, and wheezing are indicative of anaphylaxis. If the client experiences an anaphylactic reaction, the immediate action would be to maintain a patent airway. The client then would receive epinephrine. Corticosteroids may also be prescribed. The client will need to be instructed about obtaining and wearing a MedicAlert bracelet, but this is not the immediate action.

The nursing educator has just completed a lecture to a group of nurses regarding care of the client with a burn injury. A major aspect of the lecture was care of the client at the scene of a fire. Which statement, if made by a nurse, indicates a need for further instruction? 1. "Flames should be doused with water." 2. "The client should be maintained in a standing position." 3. "Flames may be extinguished by rolling the client on the ground." 4. "Flames may be smothered by the use of a blanket or another cover."

Rationale: 2 The client should be placed or maintained in a supine position; otherwise, flames may spread to other parts of the body, causing more extensive injury. Flame burns may be extinguished by rolling the client on the ground, smothering the flames with a blanket or other cover, or dousing the flames with water.

A client who attempted suicide by hanging is brought to the emergency department by emergency medical services. Which is the immediate nursing action? 1. Take the client's vital signs, including pulse oximetry reading. 2. Assess the client's respiratory status and for the presence of neck injuries. 3. Perform a focused assessment, paying particular attention to the client's neurological status. 4. Call the mental health crisis team and notify them that a client who attempted suicide is being admitted.

Rationale: 2 The immediate nursing action for a client who attempted suicide is to assess physiological status. Airway is always the priority. Therefore, assessing the client's respiratory status and for the presence of neck injuries is the immediate action that the nurse takes. Although the remaining options identify appropriate nursing actions, they are not the priority.

The police arrive at the emergency department with a client who has lacerated both wrists. Which is the initial nursing action? 1. Administer an antianxiety agent. 2. Assess and treat the wound sites. 3. Secure and record a detailed history. 4. Encourage and assist the client to ventilate feelings.

Rationale: 2 The initial nursing action is to assess and treat the self-inflicted injuries. Injuries from lacerated wrists can lead to a life-threatening situation. Other interventions, such as options 1, 3, and 4, may follow after the client has been treated medically.

A client in cardiogenic shock has a pulmonary artery catheter (Swan-Ganz type) placed. The nurse would interpret which cardiac output (CO) and pulmonary capillary wedge pressure (PCWP) readings as indicating that the client is most unstable? 1. CO 5 L/min, PCWP low 2. CO 3 L/min, PCWP low 3. CO 4 L/min, PCWP high 4. CO 3 L/min, PCWP high

Rationale: 2 The normal cardiac output is 4 to 7 L/min. With cardiogenic shock, the CO falls below normal because of failure of the heart as a pump. The PCWP, however, rises because it is a reflection of the left ventricular end-diastolic pressure, which rises with pump failure.

The nurse is caring for a client who sustained a thermal burn caused by the inhalation of steam 24 hours ago. The nurse determines that the priority nursing action is to assess which item? 1. Pain level 2. Lung sounds 3. Ability to swallow 4. Laboratory results

Rationale: 2 The priority nursing action would be to assess lung sounds. Thermal burns to the lower airways can occur with the inhalation of steam or explosive gases or with the aspiration of scalding liquids. Thermal burns to the upper airways are more common and cause erythema and edema of the airways and mucosal blisters or ulcerations. The mucosal edema can lead to upper airway obstruction, particularly during the first 24 to 48 hours after burn injury.

Which client situation is most appropriate for the nurse to consult with the Rapid Response Team (RRT)? 1.A 56-year-old client, fourth hospital day after coronary artery bypass procedure, sore chest, pain with walking, temperature 97°F (36.1°C), heart rate 84 beats/min, respirations 22 breaths/min, blood pressure 122/78 mm Hg, bored with hospitalization 2.A 45-year-old client, 2 years after kidney transplant, second hospital day for treatment of pneumonia, no urine output for 6 hours, temperature 101.4°F (38.6°C), heart rate 98 beats/min, respirations 20 breaths/min, blood pressure 168/94 mm Hg 3.A 72-year-old client, 24 hours after removal of a chest tube that was used to drain pleural fluid (effusion), temperature 97.8°F (36.6°C), heart rate 92 beats/min, respirations 28 breaths/min, blood pressure 136/86 mm Hg, anxious about going home 4.An 86-year-old client, 48 hours after operative repair of fractured hip, alert, oriented, using patient-controlled analgesia pump, temperature 96.8°F (36°C), heart rate 60 beats/min, respirations 16 breaths/min, blood pressure 120/82 mm Hg, talking with daughter

Rationale: 2 The role of an RRT is to provide internal consultative services to staff nurses to detect client problems early. Absence of urine output and temperature and blood pressure elevation describe a client who may be rejecting a transplanted kidney. The constellation of symptoms described indicates possible rejection. Internal consultation could validate that assessment. The remaining options indicate expected characteristics of the clients described and provide no indication of need for RRT consultation.

A client with a history of gastric ulcer complains of a sudden, sharp, severe pain in the midepigastric area, which then spreads over the entire abdomen. The client's abdomen is rigid and boardlike on palpation, and the client obtains most comfort from lying in the knee-chest position. The nurse suspects which condition and should perform which action? 1. Obstruction; call the operating room. 2. Perforation; notify the health care provider. 3. Intractability; administer cleansing enema. 4. Hemorrhage; increase intravenous fluid rate.

Rationale: 2 The signs and symptoms described in the question are consistent with perforation of the ulcer, which may progress to peritonitis if the perforation is large enough. A client with intestinal obstruction would most likely complain of abdominal pain and distention and nausea and vomiting. Intractability is a term that refers to continued manifestations of a disease process despite ongoing medical treatment. A client with hemorrhage would vomit blood or coffee-ground material or would expel black, tarry, or bloody stools.

A client who suffered carbon monoxide poisoning from working on an automobile in a closed garage has a carbon monoxide level of 15%. The nurse should anticipate observing which sign or symptom? 1. Coma 2. Flushing 3. Dizziness 4. Tachycardia

Rationale: 2 The signs and symptoms worsen as the carbon monoxide level rises in the bloodstream. Impaired visual acuity occurs at 5% to 10%, whereas flushing and headache are seen at 11% to 20%. Nausea and impaired dexterity appear at levels of 21% to 30%, and a 31% to 40% level is accompanied by vomiting, dizziness, and syncope. Levels of 41% to 50% cause tachypnea and tachycardia, and those higher than 50% result in coma and death.

The nurse is caring for a client who is receiving feedings by nasogastric tube. The client suddenly begins to vomit, and the nurse quickly repositions the client. The client is coughing and having difficulty breathing. What is the nurse's priority action? 1. Call a code. 2. Suction the client. 3. Check the client's vital signs. 4. Call the health care provider (HCP).

Rationale: 2 This client is at high risk for aspiration due to vomiting with a nasogastric tube in place. If the client aspirates a feeding, the nurse should immediately suction the client's airway. The client's respiratory status will then be monitored closely until a normal respiratory pattern resumes. The question presents no data indicating the need to call a code. The client's vital signs may need to be monitored, but this is not the priority action. Although the HCP may need to be notified, ensuring a patent airway is the priority.

A depressed client is found unconscious on the floor in the dayroom of a health care facility. The nurse finds several empty bottles of a prescribed tricyclic antidepressant lying near the client. Which is the priority action of the nurse? 1. Call the Poison Control Center. 2. Call the emergency response team. 3. Determine the exact number of pills taken. 4. Induce vomiting and notify the health care provider.

Rationale: 2 Tricyclic antidepressants can be fatal when taken as an overdose, regardless of the amount ingested. Life-threatening symptoms can develop after an overdose. Immediate emergency medical attention and cardiac monitoring are necessary with an overdose of tricyclic antidepressants. Options that delay immediate intervention would not be the priority actions. Vomiting is not induced in an unconscious client.

When performing a surgical dressing change of a client's abdominal dressing, the nurse notes an increase in the amount of drainage and separation of the incision line. The underlying tissue is visible to the nurse. What should the nurse do next? 1. Apply a sterile dressing soaked with povidone-iodine. 2. Apply a sterile dressing soaked with normal saline. 3. Irrigate the wound, and apply a dry sterile dressing. 4. Leave the incision exposed to the air to dry the area.

Rationale: 2 Wound dehiscence is the separation of wound edges at the suture line. Signs and symptoms include increased drainage and the exposure of underlying tissues. These usually occur 6 to 8 days after surgery. The client should be instructed to remain quiet and avoid coughing or straining. The client should be positioned to prevent further stress on the wound. Sterile dressings soaked with sterile normal saline should be used to cover the wound. The health care provider must be notified after applying this initial dressing to the wound.

A client is brought to the emergency department with partial-thickness burns to his face, neck, arms, and chest after trying to put out a car fire. The nurse should implement which nursing actions for this client? Select all that apply. 1. Restrict fluids. 2. Assess for airway patency. 3. Administer oxygen as prescribed. 4. Place a cooling blanket on the client. 5. Elevate extremities if no fractures are present. 6. Prepare to give oral pain medication as prescribed.

Rationale: 2,3 The primary goal for a burn injury is to maintain a patent airway, administer intravenous (IV) fluids to prevent hypovolemic shock, and preserve vital organ functioning. Therefore, the priority actions are to assess for airway patency and maintain a patent airway. The nurse then prepares to administer oxygen. Oxygen is necessary to perfuse vital tissues and organs. An IV line should be obtained and fluid resuscitation started. The extremities are elevated to assist in preventing shock and decrease fluid moving to the extremities, especially in the burn-injured upper extremities. The client is kept warm since the loss of skin integrity causes heat loss. The client is placed on NPO (nothing by mouth) status because of the altered gastrointestinal function that occurs as a result of a burn injury.

The nurse is assessing a client hospitalized with acute pericarditis. The nurse monitors the client for cardiac tamponade, knowing that which signs are associated with this complication of pericarditis? Select all that apply. 1. Bradycardia 2. Pulsus paradoxus 3. Distant heart sounds 4. Falling blood pressure (BP) 5. Distended jugular veins

Rationale: 2,3,4,5 Assessment findings with cardiac tamponade include tachycardia, distant or muffled heart sounds, jugular vein distention, and a falling BP, accompanied by pulsus paradoxus (a drop in inspiratory BP by more than 10 mm Hg).

The nurse is caring for a client recovering from a subtotal thyroidectomy. Which supplies should be readily accessible for the care of this client? Select all that apply. 1. Tourniquet 2. Suction supplies 3. Calcium gluconate 4. Prefilled syringe of 50% glucose 5. Tracheostomy tube insertion set

Rationale: 2,3,5 Although not common, airway obstruction after thyroid surgery is an emergency situation. Therefore, oxygen, suction equipment, calcium gluconate (to treat tetany if it occurs), and a tracheostomy tube insertion set should be readily available in the client's room. These items will be needed to treat this emergency situation. Therefore, options 2, 3, and 5 are correct. There is no reason that a tourniquet needs to be readily available; 50% glucose is used to treat severe hypoglycemia.

A client is brought to the emergency room with a snake bite to the arm. Which treatment interventions should the nurse anticipate? Select all that apply. 1. Apply ice to the site. 2. Deliver supplemental oxygen. 3. Apply a tourniquet just above the site. 4. Maintain the extremity at the level of the heart. 5. Infuse crystalloid fluids through 2 large-bore intravenous (IV) lines. 6. Immobilize the affected extremity in a position of function with a splint.

Rationale: 2,4,5,6 Interventions include giving supplemental oxygen, keeping the arm at the level of the heart, infusing crystalloid fluids through 2 large-bore IV lines, and immobilizing the arm in a position of function with a splint. Applying a tourniquet and placing ice on the area are contraindicated because they enhance the effect of the venom. Keep the person warm and provide calm reassurance. Also, apply continuous cardiac and blood pressure monitoring equipment to quickly detect clinical deterioration. Because venom can cause severe pain at the bite site, opioids are indicated. Provide tetanus prophylaxis and wound care as part of the collaborative plan of care.

The nurse is admitting a young child who arrived from the emergency department after treatment for acetaminophen overdose. After administering the antidote, the nurse should reassess the child, including which priority laboratory value? 1. Thyroid panel 2. Urine drug screen 3. Liver function panel 4. Kidney function tests

Rationale: 3 Acetaminophen overdose is harmful to the liver. Thyroid function is not affected by acetaminophen. A urine medication screen and kidney function tests may be evaluated; however, these laboratory values are not the priority concern.

A client is undergoing fluid replacement after being burned on 20% of her body 12 hours ago. The nursing assessment reveals a blood pressure of 90/50 mm Hg, a pulse rate of 110 beats/minute, and a urine output of 20 mL over the past hour. The nurse reports the findings to the health care provider (HCP) and anticipates which prescription? 1. Transfusing 1 unit of packed red blood cells 2. Administering a diuretic to increase urine output 3. Increasing the amount of intravenous (IV) lactated Ringer's solution administered per hour 4. Changing the IV lactated Ringer's solution to one that contains 5% dextrose in water

Rationale: 3 Fluid management during the first 24 hours following a burn injury generally includes the infusion of (usually) lactated Ringer's solution. Lactated Ringer's solution is an isotonic solution that contains electrolytes that will maintain fluid volume in the circulation. Fluid resuscitation is determined by urine output and hourly urine output should be at least 30 mL/hour. The client's urine output is indicative of insufficient fluid resuscitation, which places the client at risk for inadequate perfusion of the brain, heart, kidneys, and other body organs. Therefore, the HCP would prescribe an increase in the amount of IV lactated Ringer's solution administered per hour. There is nothing in the situation that calls for blood resplacement, which is not used for fluid therapy for burn injuries. Administering a diuretic would not correct the problem because fluid replacement is needed. Diuretics promote the removal of the circulating volume, thereby further compromising the inadequate tissue perfusion. Intravenous 5% dextrose solution is isotonic before administered but is hypotonic once the dextrose is metabolized. Hypotonic solutions are not appropriate for fluid resuscitation of a client with significant burn injuries.

The nurse is reviewing the medical record of a client transferred to the medical unit from the critical care unit. The nurse notes that the client received intra-aortic balloon pump (IABP) therapy while in the critical care unit. The nurse suspects that the client received this therapy for which condition? 1. Heart failure 2. Pulmonary edema 3. Cardiogenic shock 4. Aortic insufficiency

Rationale: 3 IABP therapy most often is used in the treatment of cardiogenic shock and is most effective if instituted early in the course of treatment. Use of IABP therapy is contraindicated in clients with aortic insufficiency and thoracic and abdominal aneurysms. This therapy is not used in the treatment of congestive heart failure or pulmonary edema.

The nurse receives a telephone call from a neighbor, who states that her 3-year-old child was found sitting on the kitchen floor with an empty bottle of liquid furniture polish. The mother of the child tells the nurse that the bottle was half full, that the child's breath smells like the polish, and that spilled polish is present on the front of the child's shirt. What should the nurse tell the mother to do first? 1. Call the pediatrician. 2. Induce vomiting immediately. 3. Call the Poison Control Center. 4. Bring the child to the emergency department.

Rationale: 3 If a poisoning occurs, the Poison Control Center should be contacted immediately. Calling the pediatrician would not be the immediate action because this would delay treatment. In addition, the pediatrician would immediately make a referral to the Poison Control Center. Vomiting should not be induced if the victim is unconscious or the substance ingested was a strong corrosive or a petroleum-based product. The Poison Control Center may advise the mother to bring the child to the emergency department, and if this is the case, the mother should call an ambulance.

The low-pressure alarm sounds on a ventilator. The nurse assesses the client and then attempts to determine the cause of the alarm. If unsuccessful in determining the cause of the alarm, the nurse should take what initial action? 1. Administer oxygen. 2. Check the client's vital signs. 3. Ventilate the client manually. 4. Start cardiopulmonary resuscitation.

Rationale: 3 If at any time an alarm is sounding and the nurse cannot quickly ascertain the problem, the client is disconnected from the ventilator and manual resuscitation is used to support respirations until the problem can be corrected. No reason is given to begin cardiopulmonary resuscitation. Checking vital signs is not the initial action. Although oxygen is helpful, it will not provide ventilation to the client.

The nurse is caring for a client following enucleation and notes the presence of bright red drainage on the dressing. Which most appropriate action should the nurse take at this time? 1. Document the finding. 2. Continue to monitor the drainage. 3. Notify the health care provider (HCP). 4. Mark the drainage on the dressing and monitor for any increase in bleeding.

Rationale: 3 If the nurse notes the presence of bright red drainage on the dressing, it must be reported to the HCP, because this indicates hemorrhage. Options 1, 2, and 4 are inappropriate at this time.

The nurse is providing care for a client who sustained burns over 30% of the body from a fire. On assessment, the nurse notes that the client is edematous in both burned and unburned body areas. The client's wife asks why her husband "looks so swollen." What is the nurse's best response? 1. "Constricted blood vessels have caused a loss of protein in the blood." 2. "Leaking blood vessels have led to increased protein amounts in the blood." 3. "Leaking blood vessels have led to decreased protein amounts in the blood." 4. "Constricted blood vessels have led to increased protein amounts in the blood."

Rationale: 3 In extensive burn injuries (greater than 25% of total body surface area), the edema occurs in both burned and unburned areas as a result of the increase in capillary permeability and hypoproteinemia. Edema also may be caused by the volume and oncotic pressure effects of the large fluid resuscitation volumes required.

Which step should occur first when using an automated external defibrillator (AED)? 1. Press the shock button if indicated. 2. Place the AED in the analyze mode. 3. Apply defibrillator pads on the client. 4. Check to see that no one is touching the client.

Rationale: 3 Knowing that the device needs to be in contact with the client in order to read the heart rhythm will lead you to select the option of placing the defibrillator pads on the client as the first step. The nurse next checks to see that no one is touching the client. The nurse then places the AED in the analyze mode. Then the shock button is pressed, if indicated.

A postoperative client receives a dose of naloxone hydrochloride for respiratory depression shortly after transfer to the nursing unit from the postanesthesia care unit. After administration of the medication, the nurse should assess the client for which change? 1. Pupillary changes 2. Scattered lung wheezes 3. Sudden increase in pain 4. Sudden episodes of vomiting

Rationale: 3 Naloxone hydrochloride is an antidote to opioids, and it also may be given to postoperative clients to treat respiratory depression. When given postoperatively for respiratory depression, it may reverse the effects of analgesics. Therefore, the nurse must assess the client for a sudden increase in level of postoperative pain. The remaining options are unrelated to the administration of naloxone.

A client with angina pectoris is experiencing chest pain that radiates down the left arm. The nurse administers a sublingual nitroglycerin tablet to the client. The client's pain is unrelieved, and the nurse determines that the client needs another nitroglycerin tablet. Which vital sign is the most important for the nurse to check before administering the medication? 1. Temperature 2. Respirations 3. Blood pressure 4. Radial pulse rate

Rationale: 3 Nitroglycerin acts directly on the smooth muscle of the blood vessels, causing relaxation and dilation. As a result, hypotension can occur. The nurse would check the client's blood pressure before administering the second nitroglycerin tablet. Although the respirations and apical pulse may be checked, these vital signs are not affected by this medication. The temperature also is not associated with administration of this medication.

Which readings obtained from a client's pulmonary artery catheter suggest that the client is in left-sided heart failure? 1. Cardiac output of 5 L/min 2. Right atrial pressure of 9 mm Hg 3. Pulmonary capillary wedge pressure (PCWP) of 20 mm Hg 4. Pulmonary artery systolic/diastolic pressures of 24/10 mm Hg

Rationale: 3 Normal PCWP ranges from 8 to 15 mm Hg. A PCWP of 20 mm Hg is elevated and corresponds to volume overload of the left ventricle, such as occurs in heart failure. Options 1, 2, and 4 are normal values.

A client is having frequent premature ventricular contractions. The nurse should place priority on assessment of which item? 1. Sensation of palpitations 2. Causative factors, such as caffeine 3. Blood pressure and oxygen saturation 4. Precipitating factors, such as infection

Rationale: 3 Premature ventricular contractions can cause hemodynamic compromise. Therefore, the priority is to monitor the blood pressure and oxygen saturation. The shortened ventricular filling time can lead to decreased cardiac output. The client may be asymptomatic or may feel palpitations. Premature ventricular contractions can be caused by cardiac disorders; states of hypoxemia; any number of physiological stressors, such as infection, illness, surgery, or trauma; and intake of caffeine, nicotine, or alcohol.

A client diagnosed with thrombophlebitis 1 day ago suddenly complains of chest pain and shortness of breath and is visibly anxious. Which complication should the nurse immediately assess the client for? 1. Pneumonia 2. Pulmonary edema 3. Pulmonary embolism 4. Myocardial infarction

Rationale: 3 Pulmonary embolism is a life-threatening complication of deep vein thrombosis and thrombophlebitis. Chest pain is the most common symptom; it is sudden in onset and may be aggravated by breathing. Other signs and symptoms include dyspnea, cough, diaphoresis, and apprehension. The conditions in the remaining options are not associated with thrombophlebitis.

A client's arterial blood gas results reveal a PaO2of 55 mm Hg. The client's admitting diagnosis is acute respiratory failure secondary to community-acquired pneumonia. What is the nurse's best action? 1. Repeat arterial blood gas testing. 2. Maintain continuous pulse oximetry. 3. Notify the health care provider (HCP). 4. Decrease the amount of oxygen administered.

Rationale: 3 Respiratory failure is defined as a PaO2 of 60 mm Hg or lower. The nurse should notify the HCP for further prescriptions. Common causes of hypoxemic respiratory failure are pneumonia, pulmonary embolism, and shock. This client should be receiving oxygen. Repeating the arterial blood gases and maintaining continuous pulse oximetry do nothing to correct the problem.

A client with no history of respiratory disease is admitted to the hospital with respiratory failure. Which results on the arterial blood gas report that are consistent with this disorder should the nurse expect to note? 1. PaO2 58 mm Hg, PaCO2 32 mm Hg 2. PaO2 60 mm Hg, PaCO2 45 mm Hg 3. PaO2 49 mm Hg, PaCO2 52 mm Hg 4. PaO2 73 mm Hg, PaCO2 62 mm Hg

Rationale: 3 Respiratory failure is described as a PaO2 of 60 mm Hg or lower and a PaCO2 of 50 mm Hg or higher in a client with no history of respiratory disease. In a client with a history of a respiratory disorder with hypercapnia, increases of 5 mm Hg or more (PaCO2) from the client's baseline are considered diagnostic.

A client receiving thrombolytic therapy with a continuous infusion of alteplase suddenly becomes extremely anxious and complains of itching. The nurse hears stridor and notes generalized urticaria and hypotension. Which nursing action is the priority? 1. Administer oxygen and protamine sulfate. 2. Cut the infusion rate in half and sit the client up in bed. 3. Stop the infusion and call for the Rapid Response Team (RRT). 4. Administer diphenhydramine and epinephrine and continue the infusion.

Rationale: 3 The client is experiencing an anaphylactic reaction. Therefore, the priority action is to stop the infusion and notify the RRT. The health care provider should be contacted once the client has been stabilized. The client may be treated with epinephrine, antihistamines, and corticosteroids as prescribed, but the infusion should not be continued.

The nurse is performing an assessment on a client who sustained circumferential burns of both legs. Which assessment would be the initial priority in caring for this client? 1. Assessing heart rate 2. Assessing respiratory rate 3. Assessing peripheral pulses 4. Assessing blood pressure (BP)

Rationale: 3 The client who receives circumferential burns to the extremities is at risk for altered peripheral circulation. The priority assessment would be to assess for peripheral pulses to ensure that adequate circulation is present. Although the respiratory rate and BP also would be assessed, the priority with a circumferential burn is assessment for the presence of peripheral pulses because the airway is not affected in this case.

The family of a client with a spinal cord injury rushes to the nursing station, saying that the client needs immediate help. On entering the room, the nurse notes that the client is diaphoretic with a flushed face and neck and is complaining of a severe headache. The pulse rate is 40 beats/minute, and the blood pressure is 230/100 mm Hg. The nurse acts quickly, suspecting that the client is experiencing which condition? 1. Spinal shock 2. Pulmonary embolism 3. Autonomic dysreflexia 4. Malignant hyperthermia

Rationale: 3 The client with a spinal cord injury is at risk for autonomic dysreflexia with an injury above the level of the seventh thoracic vertebra (T7). Autonomic dysreflexia is characterized by severe, throbbing headache; flushing of the face and neck; bradycardia; and sudden severe hypertension. Other signs include nasal stuffiness, blurred vision, nausea, and sweating. Autonomic dysreflexia is a life-threatening syndrome triggered by a noxious stimulus below the level of the injury. The data in the question are not associated with the conditions noted in the remaining options.

A client who has just suffered a large flail chest is experiencing severe pain and dyspnea. The client's central venous pressure (CVP) is rising, and the arterial blood pressure is falling. Which condition should the nurse interpret that the client is experiencing? 1. Fat embolism 2. Mediastinal shift 3. Mediastinal flutter 4. Hypovolemic shock

Rationale: 3 The client with severe flail chest will have significant paradoxical chest movement. This causes the mediastinal structures to swing back and forth with respiration. This movement can affect hemodynamics. Specifically, the client's CVP rises, the filling of the right side of the heart is impaired, and the arterial blood pressure falls. This is referred to as mediastinal flutter. The client with fat embolism experiences chest pain and dyspnea, but this condition occurs as a complication of a bone fracture. Mediastinal shift is a condition in which the structures of the mediastinum shift or move to the opposite side of the chest cavity; this may be caused by a pleural effusion or tension pneumothorax. In hypovolemic shock, the blood pressure falls and the pulse rises; this occurs following hemorrhage.

The client has developed atrial fibrillation, with a ventricular rate of 150 beats/minute. The nurse should assess the client for which associated signs and/or symptoms? 1. Flat neck veins 2. Nausea and vomiting 3. Hypotension and dizziness 4. Hypertension and headache

Rationale: 3 The client with uncontrolled atrial fibrillation with a ventricular rate more than 100 beats/minute is at risk for low cardiac output because of loss of atrial kick. The nurse assesses the client for palpitations, chest pain or discomfort, hypotension, pulse deficit, fatigue, weakness, dizziness, syncope, shortness of breath, and distended neck veins.

The nurse is performing an assessment on a client admitted to the nursing unit who has sustained an extensive burn injury involving 45% of total body surface area. When planning for fluid resuscitation, the nurse should consider that fluid shifting to the interstitial spaces is greatest during which time period? 1. Immediately after the injury 2. Within 12 hours after the injury 3. Between 18 and 24 hours after the injury 4. Between 42 and 72 hours after the injury

Rationale: 3 The maximum amount of edema in a client with a burn injury is seen between 18 and 24 hours after the injury. With adequate fluid resuscitation the transmembrane potential is restored to normal within 24 to 36 hours after the burn. The remaining options are incorrect.

The child with croup is being discharged from the hospital. The nurse provides instructions to the mother and advises the mother to bring the child to the emergency department if which occurs? 1. The child is irritable. 2. The child appears tired. 3. The child develops stridor. 4. The child takes fluids poorly.

Rationale: 3 The mother should be instructed to bring the child to the emergency department if the child develops stridor at rest, cyanosis, severe agitation or fatigue, or moderate to severe retractions or is unable to take oral fluids.

The nurse is caring for a child who was burned in a house fire. The nurse develops a plan of care for monitoring the child during the treatment for burn shock. The nurse identifies which assessment as providing the most accurate guide to determine the adequacy of fluid resuscitation? 1. Heart rate 2. Lung sounds 3. Level of consciousness 4. Amount of edema at the site of the burn injury

Rationale: 3 The sensorium, or level of consciousness, is an important guide to the adequacy of fluid resuscitation. The burn injury itself does not affect the sensorium, so the child should be alert and oriented. Any alteration in sensorium should be evaluated further. A neurological assessment would determine the level of consciousness in the child. Although the remaining options are important in the assessment of the child with a burn injury, they would not provide an accurate assessment of the adequacy of fluid resuscitation.

The nurse should report which assessment finding to the health care provider (HCP) before initiating thrombolytic therapy in a client with pulmonary embolism? 1. Adventitious breath sounds 2. Temperature of 99.4°F (37.4°C) orally 3. Blood pressure of 198/110 mm Hg 4. Respiratory rate of 28 breaths/minute

Rationale: 3 Thrombolytic therapy is contraindicated in a number of preexisting conditions in which there is a risk of uncontrolled bleeding, similar to the case in anticoagulant therapy. Thrombolytic therapy also is contraindicated in severe uncontrolled hypertension because of the risk of cerebral hemorrhage. Therefore, the nurse would report the results of the blood pressure to the HCP before initiating therapy.

The nurse notes that a client with sinus rhythm has a premature ventricular contraction that falls on the T wave of the preceding beat. The client's rhythm suddenly changes to one with no P waves, no definable QRS complexes, and coarse wavy lines of varying amplitude. How should the nurse correctly interpret this rhythm? 1. Asystole 2. Atrial fibrillation 3. Ventricular fibrillation 4. Ventricular tachycardia

Rationale: 3 Ventricular fibrillation is characterized by irregular chaotic undulations of varying amplitudes. Ventricular fibrillation has no measurable rate and no visible P waves or QRS complexes and results from electrical chaos in the ventricles.

The nurse is watching the cardiac monitor and notices that the rhythm suddenly changes. There are no P waves, the QRS complexes are wide, and the ventricular rate is regular but more than 140 beats/minute. The nurse determines that the client is experiencing which dysrhythmia? 1. Sinus tachycardia 2. Ventricular fibrillation 3. Ventricular tachycardia 4. Premature ventricular contractions

Rationale: 3 Ventricular tachycardia is characterized by the absence of P waves, wide QRS complexes (longer than 0.12 seconds), and typically a rate between 140 and 180 impulses/minute. The rhythm is regular.

Which clinical manifestations of a tension pneumothorax should be of immediate concern to the nurse? Select all that apply. 1. Bradypnea 2. Flattened neck veins 3. Decreased cardiac output 4. Hyperresonance to percussion 5. Tracheal deviation to the opposite side

Rationale: 3,4,5 Tension pneumothorax is the rapid accumulation of air in the pleural space. This causes extremely high intrapleural pressures, resulting in tension on the heart and great vessels. This can cause decreased cardiac output (tachycardia, hypotension), hyperresonance on percussion, and a tracheal shift away from the affected side. Bradypnea and flattened neck veins are incorrect because the client would have tachypnea and distended neck veins.

The health care provider (HCP) arrives on the nursing unit and deflates the esophageal balloon of a client's Sengstaken-Blakemore tube. The nurse should contact the HCP immediately if which occurs? 1. The client has some diarrhea that is bloody. 2. The client's blood pressure is 128/78 mm Hg. 3. The client complains of abdominal discomfort. 4. The client complains of nausea and vomits blood.

Rationale: 4 A Sengstaken-Blakemore tube may be used in a client with a diagnosis of cirrhosis and ruptured esophageal varices if other interventions are unsuccessful. The tube has an esophageal and a gastric balloon. The esophageal balloon exerts pressure on the ruptured esophageal varices and stops the bleeding. The pressure of the esophageal balloon is released at intervals to decrease the risk of trauma to esophageal tissues, including esophageal rupture or necrosis. When the balloon is deflated, the client may begin to bleed again from the esophageal varices; this would be manifested by vomiting of blood (hematemesis). The remaining options are not specifically associated with esophageal deflation.

The nurse is caring for a client who has overdosed on phenobarbital. The nurse anticipates which assessment finding with this client? 1. Hyperthermia 2. Hyperreflexia 3. Deep respirations 4. Shallow respirations

Rationale: 4 A client experiencing an overdose from barbiturates (such as phenobarbital) will experience shallow respirations; cold, clammy skin; weak, rapid pulse; hyporeflexia; coma; and possible death. Therefore, the remaining options are incorrect.

A child is admitted to the hospital after being seen in the emergency department with complaints of right lower quadrant abdominal pain, nausea and vomiting, fever, and chills. The health care provider (HCP) suspects appendicitis. Which assessment finding should the nurse immediately report to the HCP? 1. Decreasing oral temperature 2. Increasing complaints of pain 3. Refusal to take fluids by mouth 4. Sudden relief of abdominal pain

Rationale: 4 A sudden relief of pain from a suspected appendicitis is commonly indicative of a ruptured appendix. This places the individual at risk for peritonitis and shock. The HCP should be notified immediately because of the need to begin intravenous antibiotics to prevent further complications. Although increasing complaints of pain is a concern, the higher priority is sudden relief of pain because of the risk of peritonitis and shock. Temperature should be monitored but is not of highest priority. The child will be placed on NPO (nothing by mouth) status in anticipation of surgery; therefore, option 4 is incorrect.

A client who sustained an inhalation injury arrives in the emergency department. On initial assessment, the nurse notes that the client is very confused and combative. The nurse determines that the client is most likely experiencing which condition? 1. Pain 2. Fear 3. Anxiety 4. Hypoxia

Rationale: 4 After a burn injury, clients normally are alert. If a client becomes confused or combative, hypoxia may be the cause. Hypoxia occurs after inhalation injury and also may occur after an electrical injury. Although anxiety, fear, and pain may occur, confusion and combativeness are most likely associated with hypoxia.

The nurse is caring for a client who has overdosed on amphetamines. The nurse anticipates noting which assessment finding in this client? 1. Bradypnea 2. Bradycardia 3. Hypothermia 4. Hypertension

Rationale: 4 An overdose from amphetamines can cause agitation, increased temperature, increased pulse, increased respiratory rate, increased blood pressure, cardiac dysrhythmias, myocardial infarction, hallucinations, seizures, and possible death. Therefore, the remaining options are incorrect.

The home health nurse is visiting an older client whose family has gone out for the day. During the visit, the client experiences chest pain that is unrelieved by sublingual nitroglycerin tablets given by the nurse. Which action by the nurse would be appropriate at this time? 1. Notify a family member who is the next of kin. 2. Drive the client to the health care provider's (HCP's) office. 3. Inform the home care agency supervisor that the visit may be prolonged. 4. Call for an ambulance to transport the client to the hospital emergency department.

Rationale: 4 Chest pain that is unrelieved by rest and nitroglycerin might not be typical anginal pain but may signal myocardial infarction (MI). Because the risk of sudden cardiac death is greatest in the first 24 hours after MI, it is imperative that the client receive emergency cardiac care. Communication with the family or home care agency delays client treatment, which is needed immediately. The HCP's office is not equipped to treat MI.

The nurse is caring for a client who sustained multiple fractures in a motor vehicle crash 12 hours earlier. The client now exhibits severe dyspnea, tachycardia, and mental confusion, and the nurse suspects fat embolism. Which is the initial nursing action? 1. Reassess the vital signs. 2. Palpate bilateral peripheral pulses. 3. Perform a neurological assessment. 4. Position the client in a Fowler's position.

Rationale: 4 Clients with fractures are at risk for fat embolism. With suspected fat embolism, the nurse would position the client in a sitting (Fowler's) position to relieve dyspnea. Supplemental oxygen is indicated to reduce the signs of hypoxia. The health care provider needs to be notified. Vital signs will need to be taken, but this action may delay initial and required interventions. Peripheral pulse assessment is not a priority action. A neurological assessment needs to be performed, but this would not be the initial nursing action.

The nurse is caring for a client who sustained a burn injury to the anterior arms and anterior chest area from a fire. Which assessment finding would indicate that the client sustained a respiratory injury as a result of the burn? 1. Fear and anxiety 2. Complaints of pain 3. Clear breath sounds 4. Use of accessory muscles for breathing

Rationale: 4 Clinical indicators of respiratory injury in a burn-injured client include the presence of facial burns, the presence of soot around the mouth or nose, and singed nasal hairs. Signs of respiratory difficulty include changes in respiratory rate and the use of accessory muscles for breathing. Although anxiety may be a sign of hypoxemia, anxiety along with bradycardia, dysrhythmias, and lethargy would be more likely to indicate a concern related to a respiratory injury. Abnormal breath sounds and abnormal arterial blood gas values also would be noted.

The hospital code team is responding to a respiratory emergency of a client admitted during the night with a fractured hip and pelvis after a motor vehicle collision (MVC). The client first became confused and then developed dyspnea, chest pain, and a petechial rash on his neck, upper arms, and chest. What condition is this client at risk for? 1. Pulmonary fibrosis 2. Hypovolemic shock 3. Blood clot embolism 4. Fat embolism syndrome (FES)

Rationale: 4 FES generally happens within 12 to 48 hours after a long bone, hip, or pelvis fracture and occurs most often in young men between ages 20 and 40 years and in older adults between ages 70 and 80. FES is a serious complication in which fat globules are released from the yellow bone marrow into the bloodstream and clog small blood vessels that supply vital organs, most commonly the lungs.

The nurse has administered a dose of salmeterol to a client. The client develops a generalized rash and urticaria, and the eyelids begin to swell. Which action should the nurse take? 1. Apply a lanolin-based cream to the rash. 2. Encourage the client to drink fluids quickly. 3. Assess the client's vision with a Snellen chart. 4. Call the health care provider (HCP) immediately.

Rationale: 4 Hypersensitivity reaction can occur in clients taking salmeterol. Signs include rash; urticaria; and swelling of the face, lips, or eyelids. The nurse should call the HCP immediately if any of these occur. The other options are incorrect.

The nurse is caring for a client with a chest tube drainage system. While the client is being assisted to sit up in bed in preparation for ambulation, the chest tube accidentally disconnects from the chest drainage system. Which is the initial nursing action? 1. Contact the health care provider (HCP). 2. Call a respiratory therapist to come to the bedside. 3. Encourage the client to perform the Valsalva maneuver. 4. Place the end of the chest tube in a container of sterile water.

Rationale: 4 If a chest tube becomes disconnected, the nurse should as quickly as possible place the end of the tube in a container of sterile water until the drainage system can be replaced. It is not necessary to contact a respiratory therapist at this time. It may be necessary to contact the HCP, but this would not be the initial nursing action. Asking the client to perform a Valsalva maneuver is not appropriate and could be harmful.

A client arrives at the emergency department following a burn injury that occurred in the basement at home, and an inhalation injury is suspected. What would the nurse anticipate to be prescribed for the client? 1. 100% oxygen via an aerosol mask 2. Oxygen via nasal cannula at 6 L/minute 3. Oxygen via nasal cannula at 15 L/minute 4. 100% oxygen via a tight-fitting, nonrebreather face mask

Rationale: 4 If an inhalation injury is suspected, administration of 100% oxygen via a tight-fitting nonrebreather face mask is prescribed until carboxyhemoglobin levels fall (usually below 15%). In inhalation injuries, the oropharynx is inspected for evidence of erythema, blisters, or ulcerations. The need for endotracheal intubation also is assessed. Administration of oxygen by aerosol mask and cannula are incorrect and would not provide the necessary oxygen supply needed for adequate tissue perfusion for the client with a likely inhalation injury.

The nurse witnesses an accident whereby a pedestrian is hit by an automobile. The nurse stops at the scene and assesses the victim. The nurse notes that the victim is responsive and has suffered trauma to the thorax resulting in a flail chest involving at least 3 ribs. What is the nurse's priority action for this victim? 1. Assist the victim to sit up. 2. Remove the victim's shirt. 3. Turn the victim onto the side opposite the flail chest. 4. Apply firm but gentle pressure with the hands to the flail segment.

Rationale: 4 If flail chest is present, the nurse applies firm but gentle pressure to the flail segments of the ribs to stabilize the chest wall, which will ultimately help the victim's respiratory status. The nurse does not move an injured client for fear of worsening an undetected spinal injury. Removing the victim's shirt is of no value in this situation and could in fact result in chilling the victim, which is counterproductive. Injured clients should be kept warm until help arrives at the scene.

A client in a postpartum unit complains of sudden sharp chest pain and dyspnea. The nurse notes that the client is tachycardic and the respiratory rate is elevated. The nurse suspects a pulmonary embolism. Which should be the initial nursing action? 1. Initiate an intravenous line. 2. Assess the client's blood pressure. 3. Prepare to administer morphine sulfate. 4. Administer oxygen, 8 to 10 L/minute, by face mask.

Rationale: 4 If pulmonary embolism is suspected, oxygen should be administered, 8 to 10 L/minute, by face mask. Oxygen is used to decrease hypoxia. The client also is kept on bed rest with the head of the bed slightly elevated to reduce dyspnea. Morphine sulfate may be prescribed for the client, but this would not be the initial nursing action. An intravenous line also will be required, and vital signs need to be monitored, but these actions would follow the administration of oxygen.

The nurse in the recovery room is caring for a client who underwent neurosurgery. Sequential compression devices (SCDs) have been applied to prevent venous stasis. While awaiting client transfer to the intensive care unit, the recovery room nurse should perform which critical assessment? 1. Assess radial pulses. 2. Log roll client to check skin integrity. 3. Monitor hemoglobin and hematocrit levels. 4. Monitor vascular status of the lower extremities.

Rationale: 4 SCDs may be useful to provide circulatory assistance after surgery. The critical nursing assessment includes monitoring the vascular status of the lower extremities. The remaining options may be components of the nursing assessment, but these actions are not part of the critical assessment required with use of SCDs.

A client in sinus bradycardia, with a heart rate of 45 beats/minute, complains of dizziness and has a blood pressure of 82/60 mm Hg. Which prescription should the nurse anticipate will be prescribed? 1. Administer digoxin. 2. Defibrillate the client. 3. Continue to monitor the client. 4. Prepare for transcutaneous pacing.

Rationale: 4 Sinus bradycardia is noted with a heart rate less than 60 beats per minute. This rhythm becomes a concern when the client becomes symptomatic. Hypotension and dizziness are signs of decreased cardiac output. Transcutaneous pacing provides a temporary measure to increase the heart rate and thus perfusion in the symptomatic client. Defibrillation is used for treatment of pulseless ventricular tachycardia and ventricular fibrillation. Digoxin will further decrease the client's heart rate. Continuing to monitor the client delays necessary intervention.

A child is receiving succimer for the treatment of lead poisoning. The nurse should monitor which most important laboratory result? 1. Iron level 2. Calcium level 3. Red blood cell count 4. Blood urea nitrogen level

Rationale: 4 Succimer is a medication that is used to treat lead poisoning. Renal function (blood urea nitrogen and creatinine) is monitored closely during the administration of chelation therapy because the medication is excreted via the kidneys. Although it is important to monitor the iron level, calcium level, and red blood cell count, these results are not specific to chelation therapy, so they are not the most important lab values to monitor.

The nurse is caring for a client in the emergency department who has sustained a head injury. The client momentarily lost consciousness at the time of the injury and then regained it. The client now has lost consciousness again. The nurse takes quick action, knowing that this sequence is compatible with which most likely condition? 1. Concussion 2. Skull fracture 3. Subdural hematoma 4. Epidural hematoma

Rationale: 4 The changes in neurological signs from an epidural hematoma begin with loss of consciousness as arterial blood collects in the epidural space and exerts pressure. The client regains consciousness as the cerebrospinal fluid is reabsorbed rapidly to compensate for the rising intracranial pressure. As the compensatory mechanisms fail, even small amounts of additional blood cause the intracranial pressure to rise rapidly, and the client's neurological status deteriorates quickly.

A client with a left arm fracture supported in a cast complains of loss of sensation in the left fingers. The nursing assessment identifies pallor in the distal portion of the arm, poor capillary refill, and a diminished left radial pulse. On the basis of these findings, the nurse would take which as a priority action? 1. Apply ice to the site. 2. Document the findings. 3. Administer pain medication. 4. Contact the health care provider (HCP).

Rationale: 4 The client with limb pallor, slow capillary refill, weakened or lost pulse, and absence of sensation or motion in the distal limb may have arterial damage from a lacerated, contused, thrombosed, or severed artery. These signs also can occur with constriction from a tight cast. Regardless of the cause, the nurse notifies the HCP immediately. Emergency intervention is needed, which could entail removal of the constricting bandage, fracture reduction, or surgery to repair the area. Application of ice could exacerbate the problem. Although the nurse would document the findings, this is not the priority action. No data are presented in the question to indicate that the client needs pain medication.

A mother brings her child to the emergency department. Based on the child's sitting position, drooling, and apparent respiratory distress, a diagnosis of epiglottitis is suspected. The nurse should plan for which priority intervention? 1. Obtaining a chest x-ray 2. Asking the mother about the precipitating events 3. Obtaining weight for correct antibiotic dose infusion 4. Providing assisted ventilation and obtaining the necessary equipment

Rationale: 4 The highest priority with epiglottitis is to have assisted ventilation available because the highest risk with this child is complete airway obstruction. Therefore, interventions related to airway are the priority. Physiological interventions continue to have the highest priority, with assessment of breath and heart sounds and then obtaining pulse oximetry being priorities. Once the airway is stabilized, the temperature, weight, and a chest x-ray can be obtained. The last priority is asking about precipitating events, which is done once physiological needs are met.

The nurse is performing an assessment on a client who was admitted with a diagnosis of carbon monoxide poisoning. Which assessment performed by the nurse would primarily elicit data related to a deterioration of the client's condition? 1. Skin color 2. Apical rate 3. Respiratory rate 4. Level of consciousness

Rationale: 4 The neurological system is primarily affected by carbon monoxide poisoning. With high levels of carbon monoxide, the neurological status progressively deteriorates. Although skin color, apical rate, and respiratory rate would be components of the assessment of the client with carbon monoxide poisoning, assessment of the neurological status of the client would elicit data specific to a deterioration in the client's condition.

A pulmonary artery catheter is inserted into a client during cardiac surgery. The nurse is monitoring the right atrial pressure (RAP). Which finding requires immediate nursing intervention? 1. 4 mm Hg 2. 6 mm Hg 3. 8 mm Hg 4. 12 mm Hg

Rationale: 4 The normal RAP is 1 to 8 mm Hg; therefore, all options within this range (1, 2, and 3) are normal findings. An elevated RAP can be indicative of right ventricular failure.

A client with a probable minor head injury resulting from a motor vehicle crash is admitted to the hospital for observation. The nurse leaves the cervical collar applied to the client in place until when? 1. The family comes to visit 2. The nurse needs to do physical care 3. The health care provider makes rounds 4. The results of spinal radiography are known

Rationale: 4 There is a significant association between cervical spine injury and head injury. For this reason, the nurse leaves any form of spinal immobilization in place until spinal radiographs rule out fracture or other damage. The remaining options are incorrect.

A postpartum client with femoral thrombophlebitis has developed sudden shortness of breath and appears very anxious. What is the nurse's priority action for this client? 1. Check the client's blood pressure immediately. 2. Elevate the head of the bed to 30 to 45 degrees. 3. Initiate an intravenous line if one is not already in place. 4. Administer oxygen by face mask as per protocol at 8 to 10 L/min.

Rationale: 4 This client is at increased risk for pulmonary embolus and is exhibiting symptoms. Because pulmonary circulation is compromised in the presence of an embolus, cardiorespiratory support is initiated by oxygen administration. Although the remaining options may be implemented, none of these is the priority nursing action.

A client is brought to the emergency department immediately after a smoke inhalation injury. The nurse initially prepares the client for which treatment? 1. Pain medication 2. Endotracheal intubation 3. Oxygen via nasal cannula 4. 100% humidified oxygen by face mask

Rationale: 4 With a smoke inhalation injury, the client is immediately treated with 100% humidified oxygen delivered by face mask. This method provides a greater concentration of oxygen than oxygen delivered via nasal cannula. Endotracheal intubation is needed if the client exhibits respiratory stridor, which indicates airway obstruction. Pain medication may be needed but would not be the initial intervention.


Kaugnay na mga set ng pag-aaral

Chapter 8 Motivation and reward system management

View Set

Paper 3: History of the Americas: Civil War; causes, course, and effects (1840-1877)

View Set

Astronomy Ch. 21 TEST: Stellar Explosions

View Set

Pathology of Stroke (Cerebrovascular Disease)

View Set